SlideShare a Scribd company logo
1 of 39
Download to read offline
1 
Assignment Problems 
Key Points 
 An assignment problem is a particular case of transportation problem. 
 The assignment is to be made on a one-to-one basis (one job to one worker). 
 The objective in assignment problem is to assign certain number of resources or facilities to an equal number of tasks or activities.( examples-job to machines or workers, products to factories, salesmen to territories, contracts to bidders, etc) to minimize total cost /time or maximize profit/revenue/efficiency. 
 An assignment problem is a balanced when the number of rows is equal to number of columns. 
 An unbalanced assignment problem can be balanced by adding dummy rows or columns as the case may be to make rows equal to columns. 
 In the dummy row or column all elements are zero. 
 An assignment problem involving restrictions in allocations (worker cannot be assigned to a machine as he may not possess the skill to operate the machine) is known as prohibited assignment. 
 In case of prohibited assignment problem, we assign cost/time of higher value say M at the prohibited combination. 
 An assignment problem can have more than one solution giving the same answer. 
 Alternate (Multiple) optimum assignment solution exists, when there are multiple zeros in columns and rows. 
 Hungarian method is the most efficient method to solve assignment problems when the objective is minimization. 
 In case the objective function is maximization of say profit, we first convert the profit table into an opportunity loss table before we apply the Hungarian method. 
 A travelling salesman problem is typical assignment problem with two additional constraints: 
 Salesmen should not visit the city twice until he has visited all the cities.
2 
 No assignment should be made along the diagonal line. 
 Assignment technique is of little use to a firm whose facilities are perfect substitutes of each other. For example if there are three identical machines doing the same job, any machine can be assigned for the job and hence one-to-one allocation will be disturbed. 
Hungarian Method 
 Developed by Hungarian Mathematician D. Konig. 
 It works on the principle of reducing the given cost matrix to a matrix of opportunity cost. 
 It reduces the given cost/time or opportunity loss matrix to the extent of having one zero in each row and column. 
Steps involved in solving Assignment Problems by Hungarian Method Step1-Express the given problem into an n × n cost matrix or table. Step 2- Subtract the minimum element of each row of the matrix from all elements of the respective row. This step is known as Row Minima. Step 3- Subtract the minimum element of each column from all the elements in that respective column. This step is known as Column Minima. Step 4- Draw the least (minimum) number of horizontal and/or vertical lines to cover all zeros in the matrix. If the number of lines drawn are equal to the number of rows or columns, optimum solution has been reached. In such a case proceed to step number 7 for assignment. However if the number of lines are not equal to rows or columns, proceed to step number 5 for modification. Step 5- Select the smallest element not covered by the lines, subtract it from all uncovered elements including itself, add it to the elements which are crossed by two lines and reproduce other elements crossed by one line intact. Step6-Repeat steps 4 and 5 until an optimal solution is reached. Step 7- Proceed to job assignments as follows: 
Examine the rows one by one starting with the first row until a row with an exactly one zero is found. Mark the zero by enclosing it in a square indicating assignment of the task to the
3 
facility. After doing so, cross out all the zeros (if any) in that column as they cannot be used to make other assignments. Examine next the columns for any column having a single zero, starting from the first column. Mark the zero as mentioned above, crossing out the remaining zeros (if any) in that row. Repeat steps (i) and (ii) alternatively until either of the following conditions occur: All the zeros have been marked or crossed, ensuring in the process that each row has a zero marked. This means that optimum solution has been reached. All the zeros cannot be marked /crossed .There are at least two zeros in each row and column which cannot be marked by using the above mentioned step 7-(i) and (ii).This means that more than one solution exists. In such a case we mark any one zero in a row of our choice and cross out the remaining zeros in the row and the column where the zero is marked. Carry out this exercise till we get one zero marked or assigned in each row and there is no further assignment required. Step 8- Summarize the result by having the final assignment table. Illustrations 1. A company has five jobs to be done. The following table shows the cost of assigning each job to each machine. Assign five jobs to the five machines so as to minimize the total cost in INR in ‘000’. 
Machine 
Job 
1 
2 
3 
4 
5 
M1 
5 
11 
10 
12 
4 
M2 
2 
4 
6 
3 
5 
M3 
3 
12 
14 
6 
4 
M4 
6 
14 
4 
11 
7 
M5 
7 
9 
8 
12 
5 
Solution: Hungarian method is used to obtain optimal solution.
4 
As the numbers of rows are equal to columns, we have a balanced assignment table and move on to step number 2. Step 2- Subtract the minimum element of each row of the matrix from all elements of the respective row. This step is known as Row Minima. 
Machine 
Job 
1 
2 
3 
4 
5 
M1 
1 
7 
6 
8 
0 
M2 
0 
2 
4 
1 
3 
M3 
0 
9 
11 
3 
1 
M4 
2 
10 
0 
7 
3 
M5 
2 
4 
3 
7 
0 
Note-In the first row 4 is lowest element and we have subtracted this lowest element from all the elements in that row. The same methodology is used for the other rows. Step 3- Subtract the minimum element of each column from all the elements in that respective column. This step is known as Column Minima. 
Machine 
Job 
1 
2 
3 
4 
5 
M1 
1 
5 
6 
7 
0 
M2 
0 
0 
4 
0 
3 
M3 
0 
7 
11 
2 
1 
M4 
2 
8 
0 
6 
3 
M5 
2 
2 
3 
6 
0
5 
Note- Column 1, 3 and 5 remain the same, as we have zero as the lowest element in each of these columns. In column 2 and 4 we have 2 and 1 as the lowest elements which we have subtracted from all elements in those respective columns. Step 4- Draw the least (minimum) number of horizontal and/or vertical lines to cover all zeros in the table. 
Machine 
Job 
1 
2 
3 
4 
5 
M1 
1 
5 
6 
7 
0 
M2 
0 
0 
4 
0 
3 
M3 
0 
7 
11 
2 
1 
M4 
2 
8 
0 
6 
3 
M5 
2 
2 
3 
6 
0 
Note- We see from the above table that only four lines are sufficient to cross all zeros.This is achieved by drawing minimum number of lines (horizontal as well as vertical) with each line crossing out maximum zeros. This is the most important step in the method and there is a chance that students can make a mistake by drawing more lines to cross out all zeros than necessary. In case you draw more lines than rows or columns it is an indication that you have a mistake. Cancel the drawn lines and draw it up fresh. Use pencils to draw the lines. As numbers of lines are not equal to rows or columns optimum solution has not been reached and we move to step number 5 for modification. Step 5- Select the smallest element not covered by the lines, subtract it from all uncovered elements including itself, add it to the elements which are crossed by two lines and reproduce other elements crossed by one line intact.
6 
Machine 
Job 
1 
2 
3 
4 
5 
M1 
1 
3 
4 
5 
0 
M2 
2 
0 
4 
0 
5 
M3 
0 
5 
9 
0 
1 
M4 
4 
8 
0 
6 
5 
M5 
2 
0 
1 
4 
0 
Note- 2 is the smallest non-crossed element we have subtracted from all non-crossed elements including itself and added this smallest element 2 to the elements which are crossed by two lines (for example element 3 in the second row was crossed by two lines and hence 2 was added to the element 3 by which the new element in this row is reading 5) .All other elements which are crossed by one line remain intact (for example digit 2 in the last row remains unchanged as it is crossed by only one line). We again draw minimum number of lines crossing out all zeros in the table as shown below: 
Machine 
Job 
1 
2 
3 
4 
5 
M1 
1 
3 
4 
5 
0 
M2 
2 
0 
4 
0 
5 
M3 
0 
5 
9 
0 
1 
M4 
4 
8 
0 
6 
5 
M5 
2 
0 
1 
4 
0 
Note- We require five lines as they are minimum number lines which are required to cross out all zeros. As the number of lines is equal to rows or columns optimum solution has been reached and we move to step number 7 for assignment. Step 7- Proceed to job assignments as follows:
7 
Examine the rows one by one starting with the first row until a row with an exactly one zero is found. Mark the zero by enclosing it in a square indicating assignment of the task to the facility. After doing so, cross out all the zeros (if any) in that column as they cannot be used to make other assignments. Examine next the columns for any column having a single zero, starting from the first column. Mark the zero as mentioned above, crossing out the remaining zeros (if any) in that row. Repeat steps (i) and (ii) alternatively until all the zeros have been either assigned or crossed. 
Machine 
Job 
1 
2 
3 
4 
5 
M1 
1 
3 
4 
5 
0 
M2 
2 
0 
4 
5 
M3 
0 
5 
9 
0 
1 
M4 
4 
8 
0 
6 
5 
M5 
2 
1 
4 
0 
Note- (i) On examining, we find that since in row number one there is a single zero for assignment which we mark it by having a square around it. After doing so we check for any other zeros in that respective column for it to be crossed out. We find that a zero is therein column number five which we cross out. (ii) After doing the row exercise, we try to find a single zero in the column starting from column number one. We find that in column no one itself there is a single zero which is marked by having a square around it. After doing so let us find whether there are other zeros in that respective row .We find that there is one zero in row number three which we cross out. (iii)We again proceed to look for a single zero in a row going row by row. We find there is a single zero in row number two for assignment which we mark it by having a square around it. Again we should not forget to cross out any other zeros in that respective column. There are no zeros in that respective column which is row number three. 
0 
0 
0 
0 
0
8 
Proceed to look for a single zero in a column by going column by column. We locate it in the column number four for assignment. We mark it by having a square around it and search for any other zeros in that respective row to be crossed out. There is none to be crossed out in row number four. We continue this row column exercise till all unique zeros is marked and others crossed out. Step 8-Summary of the assignment is given in the table below: 
Machine 
Job 
Cost in INR in ‘’000’’ 
M1 
5 
4 
M2 
4 
3 
M3 
1 
3 
M4 
3 
4 
M5 
2 
9 
Total 
23 
Each of four workers A, B, C, D can do each of the four jobs I, II, III, IV. The figures within the matrix given below show the time taken by each worker in minutes to do each job. Assign the jobs to the four workers, only one to each so as to minimize total time to do all the jobs. 
Jobs 
Workers 
A 
B 
C 
D 
I 
4 
3 
12 
7 
II 
5 
4 
7 
9 
III 
3 
1 
6 
2 
IV 
5 
6 
9 
5 
Solution: Hungarian method is used to obtain optimal solution.
9 
As the numbers of rows are equal to columns, we have a balanced assignment table and move on to step number 2. Step 2- Subtract the minimum element of each row of the matrix from all elements of the respective row. This step is known as Row Minima. 
Jobs 
Workers 
A 
B 
C 
D 
I 
1 
0 
9 
4 
II 
1 
0 
3 
5 
III 
2 
0 
5 
1 
IV 
0 
1 
4 
0 
Note-In the first row 3 is lowest element and we have subtracted this lowest element from all the elements in that row. The same methodology is used for the other rows. Step 3- Subtract the minimum element of each column from all the elements in that respective column. This step is known as Column Minima. 
Jobs 
Workers 
A 
B 
C 
D 
I 
1 
0 
6 
4 
II 
1 
0 
0 
5 
III 
2 
0 
2 
1 
IV 
0 
1 
1 
0 
Step 4- Draw the least (minimum) number of horizontal and/or vertical lines to cover all zeros in the table.
10 
Note- We see that only three lines are sufficient to cancel all zeros. We have drawn minimum number of horizontal and vertical lines with each line striking off maximum zeros. As the number of lines are less than the rows and columns, we move on to step number 5 for modification. Step 5- Select the smallest element not covered by the lines, subtract it from all uncovered elements including itself, add it to the elements which are crossed by two lines and reproduce other elements crossed by one line intact. 
Jobs 
Workers 
A 
B 
C 
D 
I 
0 
0 
5 
3 
II 
1 
1 
0 
5 
III 
1 
0 
1 
0 
IV 
0 
2 
1 
0 
Note- 1 is the smallest non-crossed element we have subtracted from all non-crossed elements including itself and added this smallest element 1 to the elements which are crossed by two lines (for example element 1 in the last row was crossed by two lines and hence 1 was added to the element 1 by which the new element in this row instead is reading 2) .All other elements which are crossed by one line remain intact (for example digit 1 in the last row remains unchanged as it is crossed by only one line). Again we draw minimum number of horizontal or vertical lines crossing out all zeros. 
Jobs 
Workers 
A 
B 
C 
D 
I 
1 
0 
6 
4 
II 
1 
0 
0 
5 
III 
2 
0 
2 
1 
IV 
0 
1 
1 
0
11 
Jobs 
Workers 
A 
B 
C 
D 
I 
0 
0 
5 
3 
II 
1 
1 
0 
5 
III 
1 
0 
1 
0 
IV 
0 
2 
1 
0 
Note- We require four lines (minimum) to cross out all zeros. As the numbers of lines drawn are equal to rows and columns optimum solution has been reached and we proceed to Step 7 for assignment. 
Jobs 
Workers 
A 
B 
C 
D 
I 
0 
0 
5 
3 
II 
1 
1 
0 
5 
III 
1 
0 
1 
0 
IV 
0 
2 
1 
0 
Note- (i) On examining, we find that since in row number two there is a single zero for assignment which we mark it by having a square around it. After doing so we check for any other zeros in that respective column for it to be crossed out. There are no zeros to be crossed out. (ii) After doing the row exercise, we try to find a single zero in the column starting from column number one. We do not find a single zero in any of the columns (iii)We again proceed to look for a single zero in a row going row by row. We find there is a no single zero in any row. This situation signifies that we have more than one solution to the problem .i.e. Multiple Solutions 
0 
0 
0
12 
In such a case, we select the remaining zeros to be blocked of our choice ,remembering one think that while doing so ,we ensure that there is only one zero in each row for assignment. When we block the zero arbitrarily, any zeros in that respective column as well as row get crossed out. For solution 1 this is done by marking zero in the first row and first column by which zero in that column as well as row gets crossed out. Now we see that zero in third row and second column is free for assignment and zero in the fourth column in the same row gets crossed out. We have only one zero left in row number four and column number four which we mark. This completes the assignment and we get Solution-1 By doing this exercise, we get two optimum solutions whose assignment is different, but the total minutes to do all the jobs by the workers remain the same. Solution-1 
Job 
Worker 
Time in minutes 
I 
A 
4 
II 
C 
7 
III 
B 
1 
IV 
D 
5 
Total 
17 
Solution-2 Note-In the second solution we have marked zero in the first row second column and crossed zeros in the first row and second column. We continue this exercise as described above to get the solution which is given below: 
Jobs 
Workers 
A 
B 
C 
D 
I 
0 
0 
5 
3 
II 
1 
1 
0 
5 
III 
1 
0 
1 
0 
IV 
0 
2 
1 
0 
0 
0 
0 
0
13 
Job 
Worker 
Time in minutes 
I 
B 
3 
II 
C 
7 
III 
D 
2 
IV 
A 
5 
Total 
17 
3. The government solicits five different proposals with the intent of giving one job to each of the companies. The bid amounts in thousands of INR are given below with X denoting no bid submitted as the company does not meet the technical criteria for that job. Find the optimal assignment to companies such that the total cost is minimum? Solution: This prohibited assignment problem and can be solved by Hungarian method as the objective is to minimize total cost. 
Company 
Proposals(INR in ‘’000’’) 
1 
2 
3 
4 
5 
A 
50 
85 
100 
75 
80 
B 
80 
85 
95 
X 
90 
C 
70 
80 
85 
75 
80 
D 
X 
90 
95 
70 
85 
E 
85 
80 
90 
80 
90 
As the numbers of rows are equal to columns, we have a balanced assignment table and move on to step number 2. Step 2- Subtract the minimum element of each row of the matrix from all elements of the respective row. This step is known as Row Minima.
14 
Company 
Proposals(INR in ‘’000’’) 
1 
2 
3 
4 
5 
A 
0 
35 
50 
25 
30 
B 
0 
5 
15 
M 
10 
C 
0 
10 
15 
5 
10 
D 
M 
20 
25 
0 
15 
E 
5 
0 
10 
0 
10 
Note- M is so very high that even after subtracting small elements like 50 or 70, M remains unchanged. Step 3- Subtract the minimum element of each column from all the elements in that respective column. This step is known as Column Minima. 
Company 
Proposals(INR in ‘’000’’) 
1 
2 
3 
4 
5 
A 
0 
35 
40 
25 
20 
B 
0 
5 
5 
M 
0 
C 
0 
10 
5 
5 
0 
D 
M 
20 
15 
0 
5 
E 
5 
0 
0 
0 
0 
Step 4- Draw the least (minimum) number of horizontal and/or vertical lines to cover all zeros in the table.
15 
Note- As minimum number of horizontal and vertical lines are less than number of rows or columns, we move on to Step 5 for modification. Step 5- Select the smallest element not covered by the lines, subtract it from all uncovered elements including itself, add it to the elements which are crossed by two lines and reproduce other elements crossed by one line intact. 
Company 
Proposals(INR in ‘’000’’) 
1 
2 
3 
4 
5 
A 
30 
35 
25 
20 
B 
0 
0 
M 
0 
C 
5 
0 
5 
0 
D 
M 
15 
10 
0 
5 
E 
0 
0 
5 
5 
Note- As the number of lines drawn are equal to rows or columns, optimum solution has been reached and we proceed to Step number 7 for assignment. 
Company 
Proposals(INR in ‘’000’’) 
1 
2 
3 
4 
5 
A 
0 
35 
40 
25 
20 
B 
0 
5 
5 
M 
0 
C 
0 
10 
5 
5 
0 
D 
M 
20 
15 
0 
5 
E 
5 
0 
0 
0 
0 
10 
0 
0 
0
16 
Company 
Proposals(INR in ‘’000’’) 
1 
2 
3 
4 
5 
A 
0 
30 
35 
25 
20 
B 
0 
0 
0 
M 
0 
C 
0 
5 
0 
5 
0 
D 
M 
15 
10 
0 
5 
E 
10 
0 
0 
5 
5 
Note- (i) On examining, we find that since in row number two there is a single zero for assignment which we mark it by having a square around it. After doing so we check for any other zeros in that respective column for it to be crossed out. There are two zeros which can be crossed out. (ii) After doing the row exercise, we try to find a single zero in the column starting from column number one. We do not find a single zero in any of the columns (iii)We again proceed to look for a single zero in a row going row by row. We find that there is one zero in fourth row which we mark .After marking we find that there is no zero in that respective column which can be crossed out. Now there are multiple zeros in row as well as column. This situation signifies that we have more than one solution to the problem .i.e. Multiple Solutions In such a case, we select the remaining zeros to be blocked of our choice ,remembering one think that while doing so ,we ensure that there is only one zero in each row for assignment. When we block the zero arbitrarily, any zeros in that respective column as well as row get crossed out. By doing this exercise, we get two optimum solutions whose assignment is different, but the total proposal cost in INR in “000” remains the same. Solution -1
17 
Company 
Proposals 
INR in “000” 
A 
1 
50 
B 
2 
85 
C 
5 
80 
D 
4 
70 
E 
3 
90 
Total 
375 
In a similar way we do the arbitrarily allotment as mentioned above (Student if need be can refer to illustration 2) to get Solution-2. 
Company 
Proposals 
INR in “000” 
A 
1 
50 
B 
3 
95 
C 
5 
80 
D 
4 
70 
E 
2 
80 
Total 
375 
4. Schedule the training seminars in five working days of the week so that the number of students unable to attend is kept to the minimum. The details are as follows:
18 
Days 
Leasing (L) 
Portfolio Management (PM) 
Private Mutual Fund (PMF) 
Equity Research (ER) 
Monday 
50 
40 
60 
20 
Tuesday 
40 
30 
40 
30 
Wednesday 
60 
20 
30 
20 
Thursday 
30 
30 
20 
30 
Friday 
10 
20 
10 
30 
Solution: As the number of columns is not equal to the number of rows, this is a case of unbalanced assignment problem. Hence before proceeding ahead with the Hungarian method, we need to ensure that numbers of rows are equal to number of columns and this is done by introducing dummy column having all its elements as zero. The balanced assignment table is given below: 
Days 
Leasing (L) 
Portfolio Management (PM) 
Private Mutual Fund (PMF) 
Equity Research (ER) 
Dummy 
Monday 
50 
40 
60 
20 
0 
Tuesday 
40 
30 
40 
30 
0 
Wednesday 
60 
20 
30 
20 
0 
Thursday 
30 
30 
20 
30 
0 
Friday 
10 
20 
10 
30 
0 
As each row has a zero as minimum element, we straightaway proceed to Step3 (column minima). Step 3- Subtract the minimum element of each column from all the elements in that respective column. This step is known as Column Minima.
19 
Days 
Leasing (L) 
Portfolio Management (PM) 
Private Mutual Fund (PMF) 
Equity Research (ER) 
Dummy 
Monday 
40 
20 
50 
0 
0 
Tuesday 
30 
10 
30 
10 
0 
Wednesday 
50 
0 
20 
0 
0 
Thursday 
20 
10 
10 
10 
0 
Friday 
0 
0 
0 
10 
0 
Step 4- Draw the least (minimum) number of horizontal and/or vertical lines to cover all zeros in the table. 
Days 
Leasing (L) 
Portfolio Management (PM) 
Private Mutual Fund (PMF) 
Equity Research (ER) 
Dummy 
Monday 
40 
20 
50 
0 
0 
Tuesday 
30 
10 
30 
10 
0 
Wednesday 
50 
0 
20 
0 
0 
Thursday 
20 
10 
10 
10 
0 
Friday 
0 
0 
0 
10 
0 
Note- We see that only four lines are sufficient to cancel all zeros. We have drawn minimum number of horizontal and vertical lines with each line striking off maximum zeros. As the number of lines are less than the rows and columns, we move on to step number 5 for modification. Step 5- Select the smallest element not covered by the lines, subtract it from all uncovered elements including itself, add it to the elements which are crossed by two lines and reproduce other elements crossed by one line intact.
20 
Days 
Leasing (L) 
Portfolio Management (PM) 
Private Mutual Fund (PMF) 
Equity Research (ER) 
Dummy 
Monday 
30 
10 
40 
0 
0 
Tuesday 
20 
0 
20 
10 
0 
Wednesday 
50 
0 
20 
10 
10 
Thursday 
10 
0 
0 
10 
0 
Friday 
0 
0 
0 
20 
10 
As the number of lines is equal to rows or columns optimum solution has been reached and we move to step number 7 for assignment. Step 7- Proceed to job assignments as follows: Examine the rows one by one starting with the first row until a row with an exactly one zero is found. Mark the zero by enclosing it in a square indicating assignment of the task to the facility. After doing so, cross out all the zeros (if any) in that column as they cannot be used to make other assignments. Examine next the columns for any column having a single zero, starting from the first column. Mark the zero as mentioned above, crossing out the remaining zeros (if any) in that row. Repeat steps (i) and (ii) alternatively until all the zeros have been either assigned or crossed. 
Days 
Leasing (L) 
Portfolio Management (PM) 
Private Mutual Fund (PMF) 
Equity Research (ER) 
Dummy 
Monday 
30 
10 
40 
0 
0 
Tuesday 
20 
0 
20 
10 
0 
Wednesday 
50 
0 
20 
10 
10 
Thursday 
10 
0 
0 
10 
0 
Friday 
0 
0 
0 
20 
10 
Step 8-Summary of the assignment is given in the table below:
21 
Days 
Training Seminar 
Absenteeism of students 
Monday 
ER 
20 
Tuesday 
- 
- 
Wednesday 
PM 
20 
Thursday 
PMF 
20 
Friday 
L 
10 
Tuesday is the day off from the training program. 5. The Marketing Director of the multinational company faced with the problem assigning five senior marketing managers to six zones. From past experience he knows that the efficiency percentage by sales depends on marketing manager-zone combination given in the following table: 
Marketing Manager 
Zones 
1 
2 
3 
4 
5 
6 
A 
71 
83 
85 
80 
76 
78 
B 
79 
83 
67 
74 
72 
83 
C 
73 
70 
81 
82 
76 
89 
D 
91 
94 
84 
89 
81 
80 
E 
88 
89 
77 
87 
67 
74 
As an advisor to the company, recommend which zone should be manned by junior manager so as to maximize the overall efficiency of the company. Solution: We see two things which are different: The objective function is maximization of the efficiency of the company. 
The problem is an unbalanced transportation problem as the number of rows is not equal to number of columns.
22 
Note- Hungarian method can only be issued when the objective function is minimization. There is a deviation here as the objective function is maximization. In such a case we first convert the given assignment table (let the table be balanced or unbalanced) to an opportunity loss table by subtracting each and every element of the table from the highest element in the table which in this case is 94.By doing so the opportunity loss table is as following: 
Marketing Manager 
Zones 
1 
2 
3 
4 
5 
6 
A 
23 
11 
9 
14 
18 
16 
B 
15 
11 
27 
20 
22 
11 
C 
21 
24 
13 
12 
18 
5 
D 
3 
0 
10 
5 
13 
14 
E 
6 
5 
17 
7 
27 
20 
The above opportunity loss table is unbalanced as the number of rows is not equal to number of columns .A dummy row is added with each element in that row being zero. By doing so the balanced opportunity loss table is given as follows: 
Marketing Manager 
Zones 
1 
2 
3 
4 
5 
6 
A 
71 
83 
85 
80 
76 
78 
B 
79 
83 
67 
74 
72 
83 
C 
73 
70 
81 
82 
76 
89 
D 
91 
94 
84 
89 
81 
80 
E 
88 
89 
77 
87 
67 
74 
Dummy 
0 
0 
0 
0 
0 
0 
We now proceed to Step number 2 for row minima.
23 
Step 2- Subtract the minimum element of each row of the matrix from all elements of the respective row. This step is known as Row Minima. 
Marketing Manager 
Zones 
1 
2 
3 
4 
5 
6 
A 
14 
2 
0 
5 
9 
7 
B 
4 
0 
16 
9 
11 
0 
C 
16 
19 
8 
7 
13 
0 
D 
3 
0 
10 
5 
13 
14 
E 
1 
0 
12 
2 
22 
15 
Dummy 
0 
0 
0 
0 
0 
0 
As each column has a zero, column minima is not required and we move on to step number 4 Step 4- Draw the least (minimum) number of horizontal and/or vertical lines to cover all zeros in the table. 
Marketing Manager 
Zones 
1 
2 
3 
4 
5 
6 
A 
14 
2 
0 
5 
9 
7 
B 
4 
0 
16 
9 
11 
0 
C 
16 
19 
8 
7 
13 
0 
D 
3 
0 
10 
5 
13 
14 
E 
1 
0 
12 
2 
22 
15 
Dummy 
0 
0 
0 
0 
0 
0 
Note- We see that only four lines are sufficient to cancel all zeros. We have drawn minimum number of horizontal and vertical lines with each line striking off maximum zeros. As the
24 
number of lines are less than the rows and columns, we move on to step number 5 for modification. Step 5- Select the smallest element not covered by the lines, subtract it from all uncovered elements including itself, add it to the elements which are crossed by two lines and reproduce other elements crossed by one line intact. 
Marketing Manager 
Zones 
1 
2 
3 
4 
5 
6 
A 
14 
3 
0 
5 
9 
10 
B 
1 
0 
13 
6 
8 
0 
C 
13 
19 
5 
4 
10 
0 
D 
0 
0 
7 
2 
10 
14 
E 
0 
2 
11 
1 
21 
17 
Dummy 
0 
3 
0 
0 
0 
3 
Note- 1 is the smallest non-crossed element we have subtracted from all non-crossed elements including itself and added this smallest element 1 to the elements which are crossed by two lines (for example element 2 in the second column was crossed by two lines and hence 1 was added to the element 2 by which the new element in this column is reading 3) .All other elements which are crossed by one line remain intact (for example digit 14 in the first row remains unchanged as it is crossed by only one line). We again draw minimum number of lines crossing out all zeros in the table as shown below:
25 
Marketing Manager 
Zones 
1 
2 
3 
4 
5 
6 
A 
14 
3 
0 
5 
9 
10 
B 
1 
0 
13 
6 
8 
0 
C 
13 
19 
5 
4 
10 
0 
D 
0 
0 
7 
2 
10 
14 
E 
0 
2 
11 
1 
21 
17 
Dummy 
0 
3 
0 
0 
0 
3 
Note- We again see that only five lines are sufficient to cancel all zeros. We have drawn minimum number of horizontal and vertical lines with each line striking off maximum zeros. As the number of lines is less than the rows and columns, we again use step number 5 for modification. 
Marketing Manager 
Zones 
1 
2 
3 
4 
5 
6 
A 
14 
3 
0 
4 
8 
10 
B 
1 
0 
13 
5 
7 
0 
C 
13 
19 
5 
3 
9 
0 
D 
0 
0 
7 
1 
9 
14 
E 
0 
2 
11 
0 
20 
17 
Dummy 
1 
4 
1 
0 
0 
4 
We again draw minimum number of lines crossing out all zeros in the table as shown below
26 
Marketing Manager 
Zones 
1 
2 
3 
4 
5 
6 
A 
14 
3 
0 
4 
8 
10 
B 
1 
0 
13 
5 
7 
0 
C 
13 
19 
5 
3 
9 
0 
D 
0 
0 
7 
1 
9 
14 
E 
0 
2 
11 
0 
20 
17 
Dummy 
1 
4 
1 
0 
0 
4 
As the number of lines is equal to rows or columns optimum solution has been reached and we move to step number 7 for assignment. Step 7- Proceed to job assignments as follows: Examine the rows one by one starting with the first row until a row with an exactly one zero is found. Mark the zero by enclosing it in a square indicating assignment of the task to the facility. After doing so, cross out all the zeros (if any) in that column as they cannot be used to make other assignments. Examine next the columns for any column having a single zero, starting from the first column. Mark the zero as mentioned above, crossing out the remaining zeros (if any) in that row. Repeat steps (i) and (ii) alternatively until all the zeros have been either assigned or crossed. 
Marketing Manager 
Zones 
1 
2 
3 
4 
5 
6 
A 
14 
3 
0 
4 
8 
10 
B 
1 
0 
13 
5 
7 
0 
C 
13 
19 
5 
3 
9 
0 
D 
0 
0 
7 
1 
9 
14 
E 
0 
2 
11 
0 
20 
17 
Dummy 
1 
4 
1 
0 
0 
4
27 
Step 8-Summary of the assignment is given in the table below 
Marketing Manager 
Zone 
Efficiency 
A 
3 
85 
B 
2 
83 
C 
6 
89 
D 
1 
91 
E 
4 
87 
: Zone 5 is left out, which can be manned by a junior manager to maximize overall efficiency of the company. 6. An airline that operates seven days in a week has time table shown below. Crews must have a minimum layover of five hours between flights. Obtain the pairing of flights that minimize layover time away from home. For any given pair the crew will be based at the city that results in smaller layover. 
Delhi-Jaipur 
Flight No 
Departure 
Arrival 
101 
7 A.M. 
8 A.M. 
102 
8 A.M. 
9 A.M. 
103 
1.30 P.M. 
2.30 PM 
104 
6.30 P.M. 
7.30 PM
28 
Jaipur-Delhi 
Flight No 
Departure 
Arrival 
201 
8 A.M. 
9.15A.M. 
202 
8.30 A.M. 
9.45 AM 
203 
12 Noon 
1.15PM 
204 
5.30 P.M. 
6.45 PM 
For each pair also mention the place where the crew should be based. Solution: The illustration mentions about flight operating from Delhi-Jaipur and Jaipur-Delhi on a daily basis. We need to locate the crew for pair of flights where layover time (idle time) is the lowest. Since the objective is to minimize layover time between flights, we can use the Hungarian method. However before doing so we need to calculate the layover time for each pair of flights from Delhi to Jaipur and back with crew based at Delhi and similarly calculate the layover time for each pair of flights from Jaipur to Delhi and back with crew based at Jaipur. After getting the respective tables containing layover time, we select the lowest layover time out of the two tables for each pair of flights and by which get the lowest layover time table on which we need to carry out the Hungarian method.
29 
Table consisting of Layover time in minutes for crew based in Delhi Delhi-Jaipur 
Return Flight 
Flight Number 
Delhi-Jaipur Flights 
201 
202 
203 
204 
101 
*1440 
1470 
1680 
570 
102 
1380 
1410 
1620 
510 
103 
1050 
1080 
1290 
1620 
104 
750 
780 
990 
1320 
Note-*101 flight lands at Jaipur at 8 am. In case the crew flying on 101 wants to come back to Delhi by 201 flight which takes off from Jaipur at 8 am, the layover time between these two flights is 24 hours (8am takeoff next day from Jaipur-8am, the time it lands at Jaipur.) equals 24 hours which is 1440 minutes. Let us do one more calculation. Consider 104-202 combination. Note-104 flight lands at Jaipur at 7.30 pm and 202 flight takes off from Jaipur at 8.30 am. Therefore the layover time between this pair of flight will be 8.30 am next day and 7.30 pm earlier day, which is equal to 13 hours and in minutes 13×60= 780 minutes. Note-We have to keep in mind that the minimum layover time has to be five hours. Layover time between pair of flights is equal to the difference between takeoff of the return flight and the landing of the earlier flight. Further we avoid fractions in terms of hours we have taken minutes as the basis for the formation of the above table. Can you do the balance calculations? I am sure you will. Let us do the same exercise of calculating layover time in minutes for crew based in Jaipur for the Jaipur-Delhi sector.
30 
Table consisting of Layover time in minutes for crew based in Jaipur Delhi-Jaipur 
Jaipur-Delhi 
Flight Number 
Return Flights 
201 
202 
203 
204 
101 
1305 
**1275 
1065 
735 
102 
1365 
1335 
1125 
795 
103 
1695 
1665 
1455 
1125 
104 
555 
525 
315 
1425 
Note-** 202 flight lands at Delhi at 9.45 am and in case the crew on this flight wants to come back by 101 flight which departs from Delhi at 7am,the difference works out to 21 hours plus 15 minutes which is equal to 21×60+15 minutes=1275 minutes. As said before we now get the lowest layover time table from the above two tables for each pair of flights an which is as given below: 
Flights 
Flight Number 
201 
202 
203 
204 
101 
1305 
1275 
1065 
570 
102 
1365 
1335 
1125 
510 
103 
1050 
1080 
1290 
1125 
104 
555 
525 
315 
1320 
 means crew is based at Jaipur and if there is nomeans crew is based at Delhi. As the above table is balanced and least cost table, we can use the Hungarian method and proceed to Step number 2 for Row Minima. Step 2- Subtract the minimum element of each row of the matrix from all elements of the respective row. This step is known as Row Minima.
31 
Flights 
Flight Number 
201 
202 
203 
204 
101 
735 
705 
495 
0 
102 
855 
825 
615 
0 
103 
0 
30 
240 
75 
104 
240 
210 
0 
1005 
Step 3- Subtract the minimum element of each column from all the elements in that respective column. This step is known as Column Minima. 
Flights 
Flight Number 
201 
202 
203 
204 
101 
735 
675 
495 
0 
102 
855 
795 
615 
0 
103 
0 
0 
240 
75 
104 
240 
180 
0 
1005 
Step4-Draw the least (minimum) number of horizontal and/or vertical lines to cover all zeros in the table. 
Flights 
Flight Number 
201 
202 
203 
204 
101 
735 
675 
495 
0 
102 
855 
795 
615 
0 
103 
0 
0 
240 
75 
104 
240 
180 
0 
1005
32 
As the number of lines are less than the rows and columns, we move on to step number 5 for modification. Step 5- Select the smallest element not covered by the lines, subtract it from all uncovered elements including itself, add it to the elements which are crossed by two lines and reproduce other elements crossed by one line intact. 
Flights 
Flight Number 
201 
202 
203 
204 
101 
240 
160 
0 
0 
102 
360 
300 
120 
0 
103 
0 
0 
240 
570 
104 
240 
180 
0 
1500 
Note- We again see that only four lines are sufficient to cancel all zeros. We have drawn minimum number of horizontal and vertical lines with each line striking off maximum zeros. As the number of lines is less than the rows and columns, we again use step number 5 for modification 
Flights 
Flight Number 
201 
202 
203 
204 
101 
80 
0 
0 
0 
102 
200 
140 
120 
0 
103 
0 
0 
400 
730 
104 
240 
180 
0 
1500 
We again draw minimum number of lines crossing out all zeros in the table as shown below
33 
Flights 
Flight Number 
201 
202 
203 
204 
101 
80 
0 
0 
0 
102 
200 
140 
120 
0 
103 
0 
0 
400 
730 
104 
240 
180 
0 
1500 
As the number of lines is equal to rows or columns optimum solution has been reached and we move to step number 7 for assignment. 
Flights 
Flight Number 
201 
202 
203 
204 
101 
80 
0 
0 
0 
102 
200 
140 
120 
0 
103 
0 
0 
400 
730 
104 
240 
180 
0 
1500 
Step 7- Proceed to job assignments as follows: Examine the rows one by one starting with the first row until a row with an exactly one zero is found. Mark the zero by enclosing it in a square indicating assignment of the task to the facility. After doing so, cross out all the zeros (if any) in that column as they cannot be used to make other assignments. Examine next the columns for any column having a single zero, starting from the first column. Mark the zero as mentioned above, crossing out the remaining zeros (if any) in that row. Repeat steps (i) and (ii) alternatively until all the zeros have been either assigned or crossed.
34 
Flights 
Flight Number 
201 
202 
203 
204 
101 
240 
160 
0 
0 
102 
360 
300 
120 
0 
103 
0 
0 
240 
570 
104 
240 
180 
0 
1500 
Step 8-Summary of the assignment is given in the table below 
Pair of Flights 
Crew based at 
Layover Time in minutes 
101-202 
Jaipur 
1275 
102-204 
Delhi 
510 
103-201 
Delhi 
1050 
104-203 
Jaipur 
315 
Total 
3150 
Note-For pair of flight101-202 we see from the least layover time table, the crew is based at Jaipur. Remember the marking and similarly for the other pair of flight we can know where the crew will be based. 7. A travelling salesman has to visit five cities. He wishes to start from a particular city, visit each city once and then return to his starting point. The travelling cost (Rs in 000) of each city from a particular city is given below:
35 
From City 
To City 
A 
B 
C 
D 
E 
A 
M 
2 
5 
7 
1 
B 
6 
M 
3 
8 
2 
C 
8 
7 
M 
4 
7 
D 
12 
4 
6 
M 
5 
E 
1 
3 
2 
8 
M 
What is the sequence of visit of the salesman to achieve the least cost route? Solution: The given assignment table is a balance assignment table and is regarding travelling salesman. We will use Hungarian method with a difference to satisfy the condition that, the salesman should visit each city before returning to the city where he started from. Note- A travelling salesman problem is typical assignment problem with two additional constraints: Salesmen should not visit the city twice until he has visited all the cities. No assignment should be made along the diagonal line. As the numbers of rows are equal to columns, we have a balanced assignment table and move on to step number 2. Step 2- Subtract the minimum element of each row of the matrix from all elements of the respective row. This step is known as Row Minima.
36 
From City 
To City 
A 
B 
C 
D 
E 
A 
M 
1 
4 
6 
0 
B 
4 
M 
1 
6 
0 
C 
4 
3 
M 
0 
3 
D 
8 
0 
2 
M 
1 
E 
0 
2 
1 
7 
M 
Step 3- Subtract the minimum element of each column from all the elements in that respective column. This step is known as Column Minima. 
From City 
To City 
A 
B 
C 
D 
E 
A 
M 
1 
3 
6 
0 
B 
4 
M 
0 
6 
0 
C 
4 
3 
M 
0 
3 
D 
8 
0 
1 
M 
1 
E 
0 
2 
0 
7 
M 
Step 4- Draw the least (minimum) number of horizontal and/or vertical lines to cover all zeros in the table.
37 
From City 
To City 
A 
B 
C 
D 
E 
A 
M 
1 
3 
6 
0 
B 
4 
M 
0 
6 
0 
C 
4 
3 
M 
0 
3 
D 
8 
0 
1 
M 
1 
E 
0 
2 
0 
7 
M 
As the number of lines is equal to rows or columns optimum solution has been reached and we move to step number 7 for assignment. Step 7- Proceed to job assignments as follows: Examine the rows one by one starting with the first row until a row with an exactly one zero is found. Mark the zero by enclosing it in a square indicating assignment of the task to the facility. After doing so, cross out all the zeros (if any) in that column as they cannot be used to make other assignments. Examine next the columns for any column having a single zero, starting from the first column. Mark the zero as mentioned above, crossing out the remaining zeros (if any) in that row. Repeat steps (i) and (ii) alternatively until all the zeros have been either assigned or crossed. 
From City 
To City 
A 
B 
C 
D 
E 
A 
M 
1 
3 
6 
0 
B 
4 
M 
0 
6 
0 
C 
4 
3 
M 
0 
3 
D 
8 
0 
1 
M 
1 
E 
0 
2 
0 
7 
M
38 
However the solution gives the sequence A-E-A .This does not satisfy the condition that the salesman has to visit each city once before returning to the starting point. Hence we have to look at the next best solution which satisfies the above mentioned condition. This can be obtained by the next (non-zero) minimum element i.e. 1 into the solution. In the above table cost 1 occurs at three different places .Therefore we consider all these three one’s, one by one until the acceptable solution or next best solution is found to meet the above condition. Case1- We select 1 in the cell AB instead of zero assignment in the cell AE and delete row A and column B. After this step we select exclusive zero in the row or column .Except at one place i.e row number four we get exclusive zero in a row or a column. After selecting lowest element 1 in this row, in cell DE we are able to satisfy the condition. The assignment table on the above basis is as follows: 
From City 
To City 
A 
B 
C 
D 
E 
A 
M 
1 
3 
6 
0 
B 
4 
M 
0 
6 
0 
C 
4 
3 
M 
0 
3 
D 
8 
0 
1 
M 
1 
E 
0 
2 
0 
7 
M 
We have not taken other two cases as we achieved the next best solution satisfying the condition. The table below gives us the total transportation cost:
39 
From City 
To City 
Travelling Cost INR 
A 
B 
2000 
B 
C 
3000 
C 
D 
4000 
D 
E 
5000 
E 
A 
1000 
Total 
15000 
Note-No feasible solution is obtained in case we had selected 1 in cell DC. 
CLOSING NOTES ON ASSIGNMENT A variety of assignment problems have been taken to give you a flavour of this topic. Students are advised to make all attempts to solve these illustrations in the CD-ROM on their own, an only when they are stuck up as a last measure refer to the solutions therein. Go step by step to achieve the optimum solution. Notes are provided in the illustrations only for guidance. You need not be a maths expert to solve the illustrations, just believe in your ability and keep a positive approach. 
The notes given in the illustrations are for guidance only. The notes and steps mentioned in this book as well as the CD-ROM should be used by the students to get a good insight on the diversity of the illustrations solved. While solving illustrations on their own, students are requested to mention the step in brief before writing the table below the same.

More Related Content

What's hot

Transportation Modelling - Quantitative Analysis and Discrete Maths
Transportation Modelling - Quantitative Analysis and Discrete MathsTransportation Modelling - Quantitative Analysis and Discrete Maths
Transportation Modelling - Quantitative Analysis and Discrete MathsKrupesh Shah
 
Deepika(14 mba5012) transportation ppt
Deepika(14 mba5012)  transportation pptDeepika(14 mba5012)  transportation ppt
Deepika(14 mba5012) transportation pptDeepika Bansal
 
Assignment Problem
Assignment ProblemAssignment Problem
Assignment ProblemAmit Kumar
 
Assignment method
Assignment methodAssignment method
Assignment methodR A Shah
 
Hungarian assignment problem
Hungarian assignment problemHungarian assignment problem
Hungarian assignment problemBrainmapsolutions
 
Transportation and transshipment problems
Transportation  and transshipment problemsTransportation  and transshipment problems
Transportation and transshipment problemsDr. Adinath Damale
 
Chapter 1 Assignment Problems (DS) (1).pptx
Chapter 1 Assignment Problems (DS) (1).pptxChapter 1 Assignment Problems (DS) (1).pptx
Chapter 1 Assignment Problems (DS) (1).pptxPriyankaLunavat
 
Assignment problem maximum
Assignment problem  maximumAssignment problem  maximum
Assignment problem maximumRafidahAli1
 
Transportation Method Operation Research
Transportation Method Operation ResearchTransportation Method Operation Research
Transportation Method Operation ResearchR A Shah
 
Transportation Problem in Operational Research
Transportation Problem in Operational ResearchTransportation Problem in Operational Research
Transportation Problem in Operational ResearchNeha Sharma
 
Solving Degenaracy in Transportation Problem
Solving Degenaracy in Transportation ProblemSolving Degenaracy in Transportation Problem
Solving Degenaracy in Transportation Problemmkmanik
 
assignment problem
assignment problemassignment problem
assignment problemSwarit Yadav
 
Algorithm for Hungarian Method of Assignment
Algorithm for Hungarian Method of AssignmentAlgorithm for Hungarian Method of Assignment
Algorithm for Hungarian Method of AssignmentRaja Adapa
 

What's hot (20)

Transportation Modelling - Quantitative Analysis and Discrete Maths
Transportation Modelling - Quantitative Analysis and Discrete MathsTransportation Modelling - Quantitative Analysis and Discrete Maths
Transportation Modelling - Quantitative Analysis and Discrete Maths
 
Deepika(14 mba5012) transportation ppt
Deepika(14 mba5012)  transportation pptDeepika(14 mba5012)  transportation ppt
Deepika(14 mba5012) transportation ppt
 
Assignment Problem
Assignment ProblemAssignment Problem
Assignment Problem
 
Assignment Problem
Assignment ProblemAssignment Problem
Assignment Problem
 
Assignment method
Assignment methodAssignment method
Assignment method
 
Sequencing
SequencingSequencing
Sequencing
 
Hungarian assignment problem
Hungarian assignment problemHungarian assignment problem
Hungarian assignment problem
 
Transportation problems
Transportation problemsTransportation problems
Transportation problems
 
Assignment problem
Assignment problemAssignment problem
Assignment problem
 
Transportation and transshipment problems
Transportation  and transshipment problemsTransportation  and transshipment problems
Transportation and transshipment problems
 
Chapter 1 Assignment Problems (DS) (1).pptx
Chapter 1 Assignment Problems (DS) (1).pptxChapter 1 Assignment Problems (DS) (1).pptx
Chapter 1 Assignment Problems (DS) (1).pptx
 
Assignment problem maximum
Assignment problem  maximumAssignment problem  maximum
Assignment problem maximum
 
Transportation Method Operation Research
Transportation Method Operation ResearchTransportation Method Operation Research
Transportation Method Operation Research
 
Transportation Problem in Operational Research
Transportation Problem in Operational ResearchTransportation Problem in Operational Research
Transportation Problem in Operational Research
 
graphical method
graphical method graphical method
graphical method
 
Solving Degenaracy in Transportation Problem
Solving Degenaracy in Transportation ProblemSolving Degenaracy in Transportation Problem
Solving Degenaracy in Transportation Problem
 
assignment problem
assignment problemassignment problem
assignment problem
 
Concept of Duality
Concept of DualityConcept of Duality
Concept of Duality
 
Vam
VamVam
Vam
 
Algorithm for Hungarian Method of Assignment
Algorithm for Hungarian Method of AssignmentAlgorithm for Hungarian Method of Assignment
Algorithm for Hungarian Method of Assignment
 

Similar to Assignment Poblems

AMA_Assignment Theory notes
AMA_Assignment Theory notesAMA_Assignment Theory notes
AMA_Assignment Theory notesCA Niraj Thapa
 
Assignment problem
Assignment problemAssignment problem
Assignment problemAbu Bashar
 
Solving ONE’S interval linear assignment problem
Solving ONE’S interval linear assignment problemSolving ONE’S interval linear assignment problem
Solving ONE’S interval linear assignment problemIJERA Editor
 
A study on solving Assignment Problem
A study on solving Assignment ProblemA study on solving Assignment Problem
A study on solving Assignment Problemvivatechijri
 
Operations research : Assignment problem (One's method) presentation
Operations research : Assignment problem (One's method) presentationOperations research : Assignment problem (One's method) presentation
Operations research : Assignment problem (One's method) presentationPankaj Kumar
 
A Comparative Analysis Of Assignment Problem
A Comparative Analysis Of Assignment ProblemA Comparative Analysis Of Assignment Problem
A Comparative Analysis Of Assignment ProblemJim Webb
 
qadm-ppt-150918102124-lva1-app6892.pdf
qadm-ppt-150918102124-lva1-app6892.pdfqadm-ppt-150918102124-lva1-app6892.pdf
qadm-ppt-150918102124-lva1-app6892.pdfHari31856
 
Quantitativeanalysisfordecisionmaking 13427543542352-phpapp02-120719222252-ph...
Quantitativeanalysisfordecisionmaking 13427543542352-phpapp02-120719222252-ph...Quantitativeanalysisfordecisionmaking 13427543542352-phpapp02-120719222252-ph...
Quantitativeanalysisfordecisionmaking 13427543542352-phpapp02-120719222252-ph...Firas Husseini
 
Presentation (1) sooraj.R.pptx
Presentation (1) sooraj.R.pptxPresentation (1) sooraj.R.pptx
Presentation (1) sooraj.R.pptxAkshay712352
 
Hungarian Method
Hungarian MethodHungarian Method
Hungarian MethodAritra7469
 
Decision Science.pdf
Decision Science.pdfDecision Science.pdf
Decision Science.pdfpandeyaman577
 
Assignment problem
Assignment problemAssignment problem
Assignment problemRafidahAli1
 
Quantitative Math
Quantitative MathQuantitative Math
Quantitative Mathkzoe1996
 
Gauss jordan elimination through pivoting
Gauss jordan elimination through pivotingGauss jordan elimination through pivoting
Gauss jordan elimination through pivotinguis
 
Hungarian Assignment Problem
Hungarian Assignment ProblemHungarian Assignment Problem
Hungarian Assignment ProblemVivekSaurabh7
 

Similar to Assignment Poblems (20)

AMA_Assignment Theory notes
AMA_Assignment Theory notesAMA_Assignment Theory notes
AMA_Assignment Theory notes
 
Assignment problem
Assignment problemAssignment problem
Assignment problem
 
Assignment problem
Assignment problemAssignment problem
Assignment problem
 
Solving ONE’S interval linear assignment problem
Solving ONE’S interval linear assignment problemSolving ONE’S interval linear assignment problem
Solving ONE’S interval linear assignment problem
 
A study on solving Assignment Problem
A study on solving Assignment ProblemA study on solving Assignment Problem
A study on solving Assignment Problem
 
Operations research : Assignment problem (One's method) presentation
Operations research : Assignment problem (One's method) presentationOperations research : Assignment problem (One's method) presentation
Operations research : Assignment problem (One's method) presentation
 
A0280115(1)
A0280115(1)A0280115(1)
A0280115(1)
 
A Comparative Analysis Of Assignment Problem
A Comparative Analysis Of Assignment ProblemA Comparative Analysis Of Assignment Problem
A Comparative Analysis Of Assignment Problem
 
qadm-ppt-150918102124-lva1-app6892.pdf
qadm-ppt-150918102124-lva1-app6892.pdfqadm-ppt-150918102124-lva1-app6892.pdf
qadm-ppt-150918102124-lva1-app6892.pdf
 
Asssignment problem
Asssignment problemAsssignment problem
Asssignment problem
 
Quantitativeanalysisfordecisionmaking 13427543542352-phpapp02-120719222252-ph...
Quantitativeanalysisfordecisionmaking 13427543542352-phpapp02-120719222252-ph...Quantitativeanalysisfordecisionmaking 13427543542352-phpapp02-120719222252-ph...
Quantitativeanalysisfordecisionmaking 13427543542352-phpapp02-120719222252-ph...
 
Presentation (1) sooraj.R.pptx
Presentation (1) sooraj.R.pptxPresentation (1) sooraj.R.pptx
Presentation (1) sooraj.R.pptx
 
Hungarian Method
Hungarian MethodHungarian Method
Hungarian Method
 
Decision Science.pdf
Decision Science.pdfDecision Science.pdf
Decision Science.pdf
 
Assignment problem
Assignment problemAssignment problem
Assignment problem
 
Quantitative Math
Quantitative MathQuantitative Math
Quantitative Math
 
Gauss jordan elimination through pivoting
Gauss jordan elimination through pivotingGauss jordan elimination through pivoting
Gauss jordan elimination through pivoting
 
Qa 2
Qa 2Qa 2
Qa 2
 
Hungarian Assignment Problem
Hungarian Assignment ProblemHungarian Assignment Problem
Hungarian Assignment Problem
 
Hungarian algorithm
Hungarian algorithmHungarian algorithm
Hungarian algorithm
 

Recently uploaded

Incoming and Outgoing Shipments in 3 STEPS Using Odoo 17
Incoming and Outgoing Shipments in 3 STEPS Using Odoo 17Incoming and Outgoing Shipments in 3 STEPS Using Odoo 17
Incoming and Outgoing Shipments in 3 STEPS Using Odoo 17Celine George
 
INTRODUCTION TO CATHOLIC CHRISTOLOGY.pptx
INTRODUCTION TO CATHOLIC CHRISTOLOGY.pptxINTRODUCTION TO CATHOLIC CHRISTOLOGY.pptx
INTRODUCTION TO CATHOLIC CHRISTOLOGY.pptxHumphrey A Beña
 
ENGLISH 7_Q4_LESSON 2_ Employing a Variety of Strategies for Effective Interp...
ENGLISH 7_Q4_LESSON 2_ Employing a Variety of Strategies for Effective Interp...ENGLISH 7_Q4_LESSON 2_ Employing a Variety of Strategies for Effective Interp...
ENGLISH 7_Q4_LESSON 2_ Employing a Variety of Strategies for Effective Interp...JhezDiaz1
 
What is Model Inheritance in Odoo 17 ERP
What is Model Inheritance in Odoo 17 ERPWhat is Model Inheritance in Odoo 17 ERP
What is Model Inheritance in Odoo 17 ERPCeline George
 
4.18.24 Movement Legacies, Reflection, and Review.pptx
4.18.24 Movement Legacies, Reflection, and Review.pptx4.18.24 Movement Legacies, Reflection, and Review.pptx
4.18.24 Movement Legacies, Reflection, and Review.pptxmary850239
 
ECONOMIC CONTEXT - PAPER 1 Q3: NEWSPAPERS.pptx
ECONOMIC CONTEXT - PAPER 1 Q3: NEWSPAPERS.pptxECONOMIC CONTEXT - PAPER 1 Q3: NEWSPAPERS.pptx
ECONOMIC CONTEXT - PAPER 1 Q3: NEWSPAPERS.pptxiammrhaywood
 
Computed Fields and api Depends in the Odoo 17
Computed Fields and api Depends in the Odoo 17Computed Fields and api Depends in the Odoo 17
Computed Fields and api Depends in the Odoo 17Celine George
 
Visit to a blind student's school🧑‍🦯🧑‍🦯(community medicine)
Visit to a blind student's school🧑‍🦯🧑‍🦯(community medicine)Visit to a blind student's school🧑‍🦯🧑‍🦯(community medicine)
Visit to a blind student's school🧑‍🦯🧑‍🦯(community medicine)lakshayb543
 
USPS® Forced Meter Migration - How to Know if Your Postage Meter Will Soon be...
USPS® Forced Meter Migration - How to Know if Your Postage Meter Will Soon be...USPS® Forced Meter Migration - How to Know if Your Postage Meter Will Soon be...
USPS® Forced Meter Migration - How to Know if Your Postage Meter Will Soon be...Postal Advocate Inc.
 
ISYU TUNGKOL SA SEKSWLADIDA (ISSUE ABOUT SEXUALITY
ISYU TUNGKOL SA SEKSWLADIDA (ISSUE ABOUT SEXUALITYISYU TUNGKOL SA SEKSWLADIDA (ISSUE ABOUT SEXUALITY
ISYU TUNGKOL SA SEKSWLADIDA (ISSUE ABOUT SEXUALITYKayeClaireEstoconing
 
Influencing policy (training slides from Fast Track Impact)
Influencing policy (training slides from Fast Track Impact)Influencing policy (training slides from Fast Track Impact)
Influencing policy (training slides from Fast Track Impact)Mark Reed
 
ANG SEKTOR NG agrikultura.pptx QUARTER 4
ANG SEKTOR NG agrikultura.pptx QUARTER 4ANG SEKTOR NG agrikultura.pptx QUARTER 4
ANG SEKTOR NG agrikultura.pptx QUARTER 4MiaBumagat1
 
Full Stack Web Development Course for Beginners
Full Stack Web Development Course  for BeginnersFull Stack Web Development Course  for Beginners
Full Stack Web Development Course for BeginnersSabitha Banu
 
DATA STRUCTURE AND ALGORITHM for beginners
DATA STRUCTURE AND ALGORITHM for beginnersDATA STRUCTURE AND ALGORITHM for beginners
DATA STRUCTURE AND ALGORITHM for beginnersSabitha Banu
 
Procuring digital preservation CAN be quick and painless with our new dynamic...
Procuring digital preservation CAN be quick and painless with our new dynamic...Procuring digital preservation CAN be quick and painless with our new dynamic...
Procuring digital preservation CAN be quick and painless with our new dynamic...Jisc
 
Inclusivity Essentials_ Creating Accessible Websites for Nonprofits .pdf
Inclusivity Essentials_ Creating Accessible Websites for Nonprofits .pdfInclusivity Essentials_ Creating Accessible Websites for Nonprofits .pdf
Inclusivity Essentials_ Creating Accessible Websites for Nonprofits .pdfTechSoup
 
ENGLISH6-Q4-W3.pptxqurter our high choom
ENGLISH6-Q4-W3.pptxqurter our high choomENGLISH6-Q4-W3.pptxqurter our high choom
ENGLISH6-Q4-W3.pptxqurter our high choomnelietumpap1
 

Recently uploaded (20)

Incoming and Outgoing Shipments in 3 STEPS Using Odoo 17
Incoming and Outgoing Shipments in 3 STEPS Using Odoo 17Incoming and Outgoing Shipments in 3 STEPS Using Odoo 17
Incoming and Outgoing Shipments in 3 STEPS Using Odoo 17
 
INTRODUCTION TO CATHOLIC CHRISTOLOGY.pptx
INTRODUCTION TO CATHOLIC CHRISTOLOGY.pptxINTRODUCTION TO CATHOLIC CHRISTOLOGY.pptx
INTRODUCTION TO CATHOLIC CHRISTOLOGY.pptx
 
ENGLISH 7_Q4_LESSON 2_ Employing a Variety of Strategies for Effective Interp...
ENGLISH 7_Q4_LESSON 2_ Employing a Variety of Strategies for Effective Interp...ENGLISH 7_Q4_LESSON 2_ Employing a Variety of Strategies for Effective Interp...
ENGLISH 7_Q4_LESSON 2_ Employing a Variety of Strategies for Effective Interp...
 
What is Model Inheritance in Odoo 17 ERP
What is Model Inheritance in Odoo 17 ERPWhat is Model Inheritance in Odoo 17 ERP
What is Model Inheritance in Odoo 17 ERP
 
4.18.24 Movement Legacies, Reflection, and Review.pptx
4.18.24 Movement Legacies, Reflection, and Review.pptx4.18.24 Movement Legacies, Reflection, and Review.pptx
4.18.24 Movement Legacies, Reflection, and Review.pptx
 
ECONOMIC CONTEXT - PAPER 1 Q3: NEWSPAPERS.pptx
ECONOMIC CONTEXT - PAPER 1 Q3: NEWSPAPERS.pptxECONOMIC CONTEXT - PAPER 1 Q3: NEWSPAPERS.pptx
ECONOMIC CONTEXT - PAPER 1 Q3: NEWSPAPERS.pptx
 
Computed Fields and api Depends in the Odoo 17
Computed Fields and api Depends in the Odoo 17Computed Fields and api Depends in the Odoo 17
Computed Fields and api Depends in the Odoo 17
 
Visit to a blind student's school🧑‍🦯🧑‍🦯(community medicine)
Visit to a blind student's school🧑‍🦯🧑‍🦯(community medicine)Visit to a blind student's school🧑‍🦯🧑‍🦯(community medicine)
Visit to a blind student's school🧑‍🦯🧑‍🦯(community medicine)
 
USPS® Forced Meter Migration - How to Know if Your Postage Meter Will Soon be...
USPS® Forced Meter Migration - How to Know if Your Postage Meter Will Soon be...USPS® Forced Meter Migration - How to Know if Your Postage Meter Will Soon be...
USPS® Forced Meter Migration - How to Know if Your Postage Meter Will Soon be...
 
ISYU TUNGKOL SA SEKSWLADIDA (ISSUE ABOUT SEXUALITY
ISYU TUNGKOL SA SEKSWLADIDA (ISSUE ABOUT SEXUALITYISYU TUNGKOL SA SEKSWLADIDA (ISSUE ABOUT SEXUALITY
ISYU TUNGKOL SA SEKSWLADIDA (ISSUE ABOUT SEXUALITY
 
FINALS_OF_LEFT_ON_C'N_EL_DORADO_2024.pptx
FINALS_OF_LEFT_ON_C'N_EL_DORADO_2024.pptxFINALS_OF_LEFT_ON_C'N_EL_DORADO_2024.pptx
FINALS_OF_LEFT_ON_C'N_EL_DORADO_2024.pptx
 
Influencing policy (training slides from Fast Track Impact)
Influencing policy (training slides from Fast Track Impact)Influencing policy (training slides from Fast Track Impact)
Influencing policy (training slides from Fast Track Impact)
 
ANG SEKTOR NG agrikultura.pptx QUARTER 4
ANG SEKTOR NG agrikultura.pptx QUARTER 4ANG SEKTOR NG agrikultura.pptx QUARTER 4
ANG SEKTOR NG agrikultura.pptx QUARTER 4
 
Full Stack Web Development Course for Beginners
Full Stack Web Development Course  for BeginnersFull Stack Web Development Course  for Beginners
Full Stack Web Development Course for Beginners
 
DATA STRUCTURE AND ALGORITHM for beginners
DATA STRUCTURE AND ALGORITHM for beginnersDATA STRUCTURE AND ALGORITHM for beginners
DATA STRUCTURE AND ALGORITHM for beginners
 
Model Call Girl in Tilak Nagar Delhi reach out to us at 🔝9953056974🔝
Model Call Girl in Tilak Nagar Delhi reach out to us at 🔝9953056974🔝Model Call Girl in Tilak Nagar Delhi reach out to us at 🔝9953056974🔝
Model Call Girl in Tilak Nagar Delhi reach out to us at 🔝9953056974🔝
 
Procuring digital preservation CAN be quick and painless with our new dynamic...
Procuring digital preservation CAN be quick and painless with our new dynamic...Procuring digital preservation CAN be quick and painless with our new dynamic...
Procuring digital preservation CAN be quick and painless with our new dynamic...
 
Inclusivity Essentials_ Creating Accessible Websites for Nonprofits .pdf
Inclusivity Essentials_ Creating Accessible Websites for Nonprofits .pdfInclusivity Essentials_ Creating Accessible Websites for Nonprofits .pdf
Inclusivity Essentials_ Creating Accessible Websites for Nonprofits .pdf
 
ENGLISH6-Q4-W3.pptxqurter our high choom
ENGLISH6-Q4-W3.pptxqurter our high choomENGLISH6-Q4-W3.pptxqurter our high choom
ENGLISH6-Q4-W3.pptxqurter our high choom
 
LEFT_ON_C'N_ PRELIMS_EL_DORADO_2024.pptx
LEFT_ON_C'N_ PRELIMS_EL_DORADO_2024.pptxLEFT_ON_C'N_ PRELIMS_EL_DORADO_2024.pptx
LEFT_ON_C'N_ PRELIMS_EL_DORADO_2024.pptx
 

Assignment Poblems

  • 1. 1 Assignment Problems Key Points  An assignment problem is a particular case of transportation problem.  The assignment is to be made on a one-to-one basis (one job to one worker).  The objective in assignment problem is to assign certain number of resources or facilities to an equal number of tasks or activities.( examples-job to machines or workers, products to factories, salesmen to territories, contracts to bidders, etc) to minimize total cost /time or maximize profit/revenue/efficiency.  An assignment problem is a balanced when the number of rows is equal to number of columns.  An unbalanced assignment problem can be balanced by adding dummy rows or columns as the case may be to make rows equal to columns.  In the dummy row or column all elements are zero.  An assignment problem involving restrictions in allocations (worker cannot be assigned to a machine as he may not possess the skill to operate the machine) is known as prohibited assignment.  In case of prohibited assignment problem, we assign cost/time of higher value say M at the prohibited combination.  An assignment problem can have more than one solution giving the same answer.  Alternate (Multiple) optimum assignment solution exists, when there are multiple zeros in columns and rows.  Hungarian method is the most efficient method to solve assignment problems when the objective is minimization.  In case the objective function is maximization of say profit, we first convert the profit table into an opportunity loss table before we apply the Hungarian method.  A travelling salesman problem is typical assignment problem with two additional constraints:  Salesmen should not visit the city twice until he has visited all the cities.
  • 2. 2  No assignment should be made along the diagonal line.  Assignment technique is of little use to a firm whose facilities are perfect substitutes of each other. For example if there are three identical machines doing the same job, any machine can be assigned for the job and hence one-to-one allocation will be disturbed. Hungarian Method  Developed by Hungarian Mathematician D. Konig.  It works on the principle of reducing the given cost matrix to a matrix of opportunity cost.  It reduces the given cost/time or opportunity loss matrix to the extent of having one zero in each row and column. Steps involved in solving Assignment Problems by Hungarian Method Step1-Express the given problem into an n × n cost matrix or table. Step 2- Subtract the minimum element of each row of the matrix from all elements of the respective row. This step is known as Row Minima. Step 3- Subtract the minimum element of each column from all the elements in that respective column. This step is known as Column Minima. Step 4- Draw the least (minimum) number of horizontal and/or vertical lines to cover all zeros in the matrix. If the number of lines drawn are equal to the number of rows or columns, optimum solution has been reached. In such a case proceed to step number 7 for assignment. However if the number of lines are not equal to rows or columns, proceed to step number 5 for modification. Step 5- Select the smallest element not covered by the lines, subtract it from all uncovered elements including itself, add it to the elements which are crossed by two lines and reproduce other elements crossed by one line intact. Step6-Repeat steps 4 and 5 until an optimal solution is reached. Step 7- Proceed to job assignments as follows: Examine the rows one by one starting with the first row until a row with an exactly one zero is found. Mark the zero by enclosing it in a square indicating assignment of the task to the
  • 3. 3 facility. After doing so, cross out all the zeros (if any) in that column as they cannot be used to make other assignments. Examine next the columns for any column having a single zero, starting from the first column. Mark the zero as mentioned above, crossing out the remaining zeros (if any) in that row. Repeat steps (i) and (ii) alternatively until either of the following conditions occur: All the zeros have been marked or crossed, ensuring in the process that each row has a zero marked. This means that optimum solution has been reached. All the zeros cannot be marked /crossed .There are at least two zeros in each row and column which cannot be marked by using the above mentioned step 7-(i) and (ii).This means that more than one solution exists. In such a case we mark any one zero in a row of our choice and cross out the remaining zeros in the row and the column where the zero is marked. Carry out this exercise till we get one zero marked or assigned in each row and there is no further assignment required. Step 8- Summarize the result by having the final assignment table. Illustrations 1. A company has five jobs to be done. The following table shows the cost of assigning each job to each machine. Assign five jobs to the five machines so as to minimize the total cost in INR in ‘000’. Machine Job 1 2 3 4 5 M1 5 11 10 12 4 M2 2 4 6 3 5 M3 3 12 14 6 4 M4 6 14 4 11 7 M5 7 9 8 12 5 Solution: Hungarian method is used to obtain optimal solution.
  • 4. 4 As the numbers of rows are equal to columns, we have a balanced assignment table and move on to step number 2. Step 2- Subtract the minimum element of each row of the matrix from all elements of the respective row. This step is known as Row Minima. Machine Job 1 2 3 4 5 M1 1 7 6 8 0 M2 0 2 4 1 3 M3 0 9 11 3 1 M4 2 10 0 7 3 M5 2 4 3 7 0 Note-In the first row 4 is lowest element and we have subtracted this lowest element from all the elements in that row. The same methodology is used for the other rows. Step 3- Subtract the minimum element of each column from all the elements in that respective column. This step is known as Column Minima. Machine Job 1 2 3 4 5 M1 1 5 6 7 0 M2 0 0 4 0 3 M3 0 7 11 2 1 M4 2 8 0 6 3 M5 2 2 3 6 0
  • 5. 5 Note- Column 1, 3 and 5 remain the same, as we have zero as the lowest element in each of these columns. In column 2 and 4 we have 2 and 1 as the lowest elements which we have subtracted from all elements in those respective columns. Step 4- Draw the least (minimum) number of horizontal and/or vertical lines to cover all zeros in the table. Machine Job 1 2 3 4 5 M1 1 5 6 7 0 M2 0 0 4 0 3 M3 0 7 11 2 1 M4 2 8 0 6 3 M5 2 2 3 6 0 Note- We see from the above table that only four lines are sufficient to cross all zeros.This is achieved by drawing minimum number of lines (horizontal as well as vertical) with each line crossing out maximum zeros. This is the most important step in the method and there is a chance that students can make a mistake by drawing more lines to cross out all zeros than necessary. In case you draw more lines than rows or columns it is an indication that you have a mistake. Cancel the drawn lines and draw it up fresh. Use pencils to draw the lines. As numbers of lines are not equal to rows or columns optimum solution has not been reached and we move to step number 5 for modification. Step 5- Select the smallest element not covered by the lines, subtract it from all uncovered elements including itself, add it to the elements which are crossed by two lines and reproduce other elements crossed by one line intact.
  • 6. 6 Machine Job 1 2 3 4 5 M1 1 3 4 5 0 M2 2 0 4 0 5 M3 0 5 9 0 1 M4 4 8 0 6 5 M5 2 0 1 4 0 Note- 2 is the smallest non-crossed element we have subtracted from all non-crossed elements including itself and added this smallest element 2 to the elements which are crossed by two lines (for example element 3 in the second row was crossed by two lines and hence 2 was added to the element 3 by which the new element in this row is reading 5) .All other elements which are crossed by one line remain intact (for example digit 2 in the last row remains unchanged as it is crossed by only one line). We again draw minimum number of lines crossing out all zeros in the table as shown below: Machine Job 1 2 3 4 5 M1 1 3 4 5 0 M2 2 0 4 0 5 M3 0 5 9 0 1 M4 4 8 0 6 5 M5 2 0 1 4 0 Note- We require five lines as they are minimum number lines which are required to cross out all zeros. As the number of lines is equal to rows or columns optimum solution has been reached and we move to step number 7 for assignment. Step 7- Proceed to job assignments as follows:
  • 7. 7 Examine the rows one by one starting with the first row until a row with an exactly one zero is found. Mark the zero by enclosing it in a square indicating assignment of the task to the facility. After doing so, cross out all the zeros (if any) in that column as they cannot be used to make other assignments. Examine next the columns for any column having a single zero, starting from the first column. Mark the zero as mentioned above, crossing out the remaining zeros (if any) in that row. Repeat steps (i) and (ii) alternatively until all the zeros have been either assigned or crossed. Machine Job 1 2 3 4 5 M1 1 3 4 5 0 M2 2 0 4 5 M3 0 5 9 0 1 M4 4 8 0 6 5 M5 2 1 4 0 Note- (i) On examining, we find that since in row number one there is a single zero for assignment which we mark it by having a square around it. After doing so we check for any other zeros in that respective column for it to be crossed out. We find that a zero is therein column number five which we cross out. (ii) After doing the row exercise, we try to find a single zero in the column starting from column number one. We find that in column no one itself there is a single zero which is marked by having a square around it. After doing so let us find whether there are other zeros in that respective row .We find that there is one zero in row number three which we cross out. (iii)We again proceed to look for a single zero in a row going row by row. We find there is a single zero in row number two for assignment which we mark it by having a square around it. Again we should not forget to cross out any other zeros in that respective column. There are no zeros in that respective column which is row number three. 0 0 0 0 0
  • 8. 8 Proceed to look for a single zero in a column by going column by column. We locate it in the column number four for assignment. We mark it by having a square around it and search for any other zeros in that respective row to be crossed out. There is none to be crossed out in row number four. We continue this row column exercise till all unique zeros is marked and others crossed out. Step 8-Summary of the assignment is given in the table below: Machine Job Cost in INR in ‘’000’’ M1 5 4 M2 4 3 M3 1 3 M4 3 4 M5 2 9 Total 23 Each of four workers A, B, C, D can do each of the four jobs I, II, III, IV. The figures within the matrix given below show the time taken by each worker in minutes to do each job. Assign the jobs to the four workers, only one to each so as to minimize total time to do all the jobs. Jobs Workers A B C D I 4 3 12 7 II 5 4 7 9 III 3 1 6 2 IV 5 6 9 5 Solution: Hungarian method is used to obtain optimal solution.
  • 9. 9 As the numbers of rows are equal to columns, we have a balanced assignment table and move on to step number 2. Step 2- Subtract the minimum element of each row of the matrix from all elements of the respective row. This step is known as Row Minima. Jobs Workers A B C D I 1 0 9 4 II 1 0 3 5 III 2 0 5 1 IV 0 1 4 0 Note-In the first row 3 is lowest element and we have subtracted this lowest element from all the elements in that row. The same methodology is used for the other rows. Step 3- Subtract the minimum element of each column from all the elements in that respective column. This step is known as Column Minima. Jobs Workers A B C D I 1 0 6 4 II 1 0 0 5 III 2 0 2 1 IV 0 1 1 0 Step 4- Draw the least (minimum) number of horizontal and/or vertical lines to cover all zeros in the table.
  • 10. 10 Note- We see that only three lines are sufficient to cancel all zeros. We have drawn minimum number of horizontal and vertical lines with each line striking off maximum zeros. As the number of lines are less than the rows and columns, we move on to step number 5 for modification. Step 5- Select the smallest element not covered by the lines, subtract it from all uncovered elements including itself, add it to the elements which are crossed by two lines and reproduce other elements crossed by one line intact. Jobs Workers A B C D I 0 0 5 3 II 1 1 0 5 III 1 0 1 0 IV 0 2 1 0 Note- 1 is the smallest non-crossed element we have subtracted from all non-crossed elements including itself and added this smallest element 1 to the elements which are crossed by two lines (for example element 1 in the last row was crossed by two lines and hence 1 was added to the element 1 by which the new element in this row instead is reading 2) .All other elements which are crossed by one line remain intact (for example digit 1 in the last row remains unchanged as it is crossed by only one line). Again we draw minimum number of horizontal or vertical lines crossing out all zeros. Jobs Workers A B C D I 1 0 6 4 II 1 0 0 5 III 2 0 2 1 IV 0 1 1 0
  • 11. 11 Jobs Workers A B C D I 0 0 5 3 II 1 1 0 5 III 1 0 1 0 IV 0 2 1 0 Note- We require four lines (minimum) to cross out all zeros. As the numbers of lines drawn are equal to rows and columns optimum solution has been reached and we proceed to Step 7 for assignment. Jobs Workers A B C D I 0 0 5 3 II 1 1 0 5 III 1 0 1 0 IV 0 2 1 0 Note- (i) On examining, we find that since in row number two there is a single zero for assignment which we mark it by having a square around it. After doing so we check for any other zeros in that respective column for it to be crossed out. There are no zeros to be crossed out. (ii) After doing the row exercise, we try to find a single zero in the column starting from column number one. We do not find a single zero in any of the columns (iii)We again proceed to look for a single zero in a row going row by row. We find there is a no single zero in any row. This situation signifies that we have more than one solution to the problem .i.e. Multiple Solutions 0 0 0
  • 12. 12 In such a case, we select the remaining zeros to be blocked of our choice ,remembering one think that while doing so ,we ensure that there is only one zero in each row for assignment. When we block the zero arbitrarily, any zeros in that respective column as well as row get crossed out. For solution 1 this is done by marking zero in the first row and first column by which zero in that column as well as row gets crossed out. Now we see that zero in third row and second column is free for assignment and zero in the fourth column in the same row gets crossed out. We have only one zero left in row number four and column number four which we mark. This completes the assignment and we get Solution-1 By doing this exercise, we get two optimum solutions whose assignment is different, but the total minutes to do all the jobs by the workers remain the same. Solution-1 Job Worker Time in minutes I A 4 II C 7 III B 1 IV D 5 Total 17 Solution-2 Note-In the second solution we have marked zero in the first row second column and crossed zeros in the first row and second column. We continue this exercise as described above to get the solution which is given below: Jobs Workers A B C D I 0 0 5 3 II 1 1 0 5 III 1 0 1 0 IV 0 2 1 0 0 0 0 0
  • 13. 13 Job Worker Time in minutes I B 3 II C 7 III D 2 IV A 5 Total 17 3. The government solicits five different proposals with the intent of giving one job to each of the companies. The bid amounts in thousands of INR are given below with X denoting no bid submitted as the company does not meet the technical criteria for that job. Find the optimal assignment to companies such that the total cost is minimum? Solution: This prohibited assignment problem and can be solved by Hungarian method as the objective is to minimize total cost. Company Proposals(INR in ‘’000’’) 1 2 3 4 5 A 50 85 100 75 80 B 80 85 95 X 90 C 70 80 85 75 80 D X 90 95 70 85 E 85 80 90 80 90 As the numbers of rows are equal to columns, we have a balanced assignment table and move on to step number 2. Step 2- Subtract the minimum element of each row of the matrix from all elements of the respective row. This step is known as Row Minima.
  • 14. 14 Company Proposals(INR in ‘’000’’) 1 2 3 4 5 A 0 35 50 25 30 B 0 5 15 M 10 C 0 10 15 5 10 D M 20 25 0 15 E 5 0 10 0 10 Note- M is so very high that even after subtracting small elements like 50 or 70, M remains unchanged. Step 3- Subtract the minimum element of each column from all the elements in that respective column. This step is known as Column Minima. Company Proposals(INR in ‘’000’’) 1 2 3 4 5 A 0 35 40 25 20 B 0 5 5 M 0 C 0 10 5 5 0 D M 20 15 0 5 E 5 0 0 0 0 Step 4- Draw the least (minimum) number of horizontal and/or vertical lines to cover all zeros in the table.
  • 15. 15 Note- As minimum number of horizontal and vertical lines are less than number of rows or columns, we move on to Step 5 for modification. Step 5- Select the smallest element not covered by the lines, subtract it from all uncovered elements including itself, add it to the elements which are crossed by two lines and reproduce other elements crossed by one line intact. Company Proposals(INR in ‘’000’’) 1 2 3 4 5 A 30 35 25 20 B 0 0 M 0 C 5 0 5 0 D M 15 10 0 5 E 0 0 5 5 Note- As the number of lines drawn are equal to rows or columns, optimum solution has been reached and we proceed to Step number 7 for assignment. Company Proposals(INR in ‘’000’’) 1 2 3 4 5 A 0 35 40 25 20 B 0 5 5 M 0 C 0 10 5 5 0 D M 20 15 0 5 E 5 0 0 0 0 10 0 0 0
  • 16. 16 Company Proposals(INR in ‘’000’’) 1 2 3 4 5 A 0 30 35 25 20 B 0 0 0 M 0 C 0 5 0 5 0 D M 15 10 0 5 E 10 0 0 5 5 Note- (i) On examining, we find that since in row number two there is a single zero for assignment which we mark it by having a square around it. After doing so we check for any other zeros in that respective column for it to be crossed out. There are two zeros which can be crossed out. (ii) After doing the row exercise, we try to find a single zero in the column starting from column number one. We do not find a single zero in any of the columns (iii)We again proceed to look for a single zero in a row going row by row. We find that there is one zero in fourth row which we mark .After marking we find that there is no zero in that respective column which can be crossed out. Now there are multiple zeros in row as well as column. This situation signifies that we have more than one solution to the problem .i.e. Multiple Solutions In such a case, we select the remaining zeros to be blocked of our choice ,remembering one think that while doing so ,we ensure that there is only one zero in each row for assignment. When we block the zero arbitrarily, any zeros in that respective column as well as row get crossed out. By doing this exercise, we get two optimum solutions whose assignment is different, but the total proposal cost in INR in “000” remains the same. Solution -1
  • 17. 17 Company Proposals INR in “000” A 1 50 B 2 85 C 5 80 D 4 70 E 3 90 Total 375 In a similar way we do the arbitrarily allotment as mentioned above (Student if need be can refer to illustration 2) to get Solution-2. Company Proposals INR in “000” A 1 50 B 3 95 C 5 80 D 4 70 E 2 80 Total 375 4. Schedule the training seminars in five working days of the week so that the number of students unable to attend is kept to the minimum. The details are as follows:
  • 18. 18 Days Leasing (L) Portfolio Management (PM) Private Mutual Fund (PMF) Equity Research (ER) Monday 50 40 60 20 Tuesday 40 30 40 30 Wednesday 60 20 30 20 Thursday 30 30 20 30 Friday 10 20 10 30 Solution: As the number of columns is not equal to the number of rows, this is a case of unbalanced assignment problem. Hence before proceeding ahead with the Hungarian method, we need to ensure that numbers of rows are equal to number of columns and this is done by introducing dummy column having all its elements as zero. The balanced assignment table is given below: Days Leasing (L) Portfolio Management (PM) Private Mutual Fund (PMF) Equity Research (ER) Dummy Monday 50 40 60 20 0 Tuesday 40 30 40 30 0 Wednesday 60 20 30 20 0 Thursday 30 30 20 30 0 Friday 10 20 10 30 0 As each row has a zero as minimum element, we straightaway proceed to Step3 (column minima). Step 3- Subtract the minimum element of each column from all the elements in that respective column. This step is known as Column Minima.
  • 19. 19 Days Leasing (L) Portfolio Management (PM) Private Mutual Fund (PMF) Equity Research (ER) Dummy Monday 40 20 50 0 0 Tuesday 30 10 30 10 0 Wednesday 50 0 20 0 0 Thursday 20 10 10 10 0 Friday 0 0 0 10 0 Step 4- Draw the least (minimum) number of horizontal and/or vertical lines to cover all zeros in the table. Days Leasing (L) Portfolio Management (PM) Private Mutual Fund (PMF) Equity Research (ER) Dummy Monday 40 20 50 0 0 Tuesday 30 10 30 10 0 Wednesday 50 0 20 0 0 Thursday 20 10 10 10 0 Friday 0 0 0 10 0 Note- We see that only four lines are sufficient to cancel all zeros. We have drawn minimum number of horizontal and vertical lines with each line striking off maximum zeros. As the number of lines are less than the rows and columns, we move on to step number 5 for modification. Step 5- Select the smallest element not covered by the lines, subtract it from all uncovered elements including itself, add it to the elements which are crossed by two lines and reproduce other elements crossed by one line intact.
  • 20. 20 Days Leasing (L) Portfolio Management (PM) Private Mutual Fund (PMF) Equity Research (ER) Dummy Monday 30 10 40 0 0 Tuesday 20 0 20 10 0 Wednesday 50 0 20 10 10 Thursday 10 0 0 10 0 Friday 0 0 0 20 10 As the number of lines is equal to rows or columns optimum solution has been reached and we move to step number 7 for assignment. Step 7- Proceed to job assignments as follows: Examine the rows one by one starting with the first row until a row with an exactly one zero is found. Mark the zero by enclosing it in a square indicating assignment of the task to the facility. After doing so, cross out all the zeros (if any) in that column as they cannot be used to make other assignments. Examine next the columns for any column having a single zero, starting from the first column. Mark the zero as mentioned above, crossing out the remaining zeros (if any) in that row. Repeat steps (i) and (ii) alternatively until all the zeros have been either assigned or crossed. Days Leasing (L) Portfolio Management (PM) Private Mutual Fund (PMF) Equity Research (ER) Dummy Monday 30 10 40 0 0 Tuesday 20 0 20 10 0 Wednesday 50 0 20 10 10 Thursday 10 0 0 10 0 Friday 0 0 0 20 10 Step 8-Summary of the assignment is given in the table below:
  • 21. 21 Days Training Seminar Absenteeism of students Monday ER 20 Tuesday - - Wednesday PM 20 Thursday PMF 20 Friday L 10 Tuesday is the day off from the training program. 5. The Marketing Director of the multinational company faced with the problem assigning five senior marketing managers to six zones. From past experience he knows that the efficiency percentage by sales depends on marketing manager-zone combination given in the following table: Marketing Manager Zones 1 2 3 4 5 6 A 71 83 85 80 76 78 B 79 83 67 74 72 83 C 73 70 81 82 76 89 D 91 94 84 89 81 80 E 88 89 77 87 67 74 As an advisor to the company, recommend which zone should be manned by junior manager so as to maximize the overall efficiency of the company. Solution: We see two things which are different: The objective function is maximization of the efficiency of the company. The problem is an unbalanced transportation problem as the number of rows is not equal to number of columns.
  • 22. 22 Note- Hungarian method can only be issued when the objective function is minimization. There is a deviation here as the objective function is maximization. In such a case we first convert the given assignment table (let the table be balanced or unbalanced) to an opportunity loss table by subtracting each and every element of the table from the highest element in the table which in this case is 94.By doing so the opportunity loss table is as following: Marketing Manager Zones 1 2 3 4 5 6 A 23 11 9 14 18 16 B 15 11 27 20 22 11 C 21 24 13 12 18 5 D 3 0 10 5 13 14 E 6 5 17 7 27 20 The above opportunity loss table is unbalanced as the number of rows is not equal to number of columns .A dummy row is added with each element in that row being zero. By doing so the balanced opportunity loss table is given as follows: Marketing Manager Zones 1 2 3 4 5 6 A 71 83 85 80 76 78 B 79 83 67 74 72 83 C 73 70 81 82 76 89 D 91 94 84 89 81 80 E 88 89 77 87 67 74 Dummy 0 0 0 0 0 0 We now proceed to Step number 2 for row minima.
  • 23. 23 Step 2- Subtract the minimum element of each row of the matrix from all elements of the respective row. This step is known as Row Minima. Marketing Manager Zones 1 2 3 4 5 6 A 14 2 0 5 9 7 B 4 0 16 9 11 0 C 16 19 8 7 13 0 D 3 0 10 5 13 14 E 1 0 12 2 22 15 Dummy 0 0 0 0 0 0 As each column has a zero, column minima is not required and we move on to step number 4 Step 4- Draw the least (minimum) number of horizontal and/or vertical lines to cover all zeros in the table. Marketing Manager Zones 1 2 3 4 5 6 A 14 2 0 5 9 7 B 4 0 16 9 11 0 C 16 19 8 7 13 0 D 3 0 10 5 13 14 E 1 0 12 2 22 15 Dummy 0 0 0 0 0 0 Note- We see that only four lines are sufficient to cancel all zeros. We have drawn minimum number of horizontal and vertical lines with each line striking off maximum zeros. As the
  • 24. 24 number of lines are less than the rows and columns, we move on to step number 5 for modification. Step 5- Select the smallest element not covered by the lines, subtract it from all uncovered elements including itself, add it to the elements which are crossed by two lines and reproduce other elements crossed by one line intact. Marketing Manager Zones 1 2 3 4 5 6 A 14 3 0 5 9 10 B 1 0 13 6 8 0 C 13 19 5 4 10 0 D 0 0 7 2 10 14 E 0 2 11 1 21 17 Dummy 0 3 0 0 0 3 Note- 1 is the smallest non-crossed element we have subtracted from all non-crossed elements including itself and added this smallest element 1 to the elements which are crossed by two lines (for example element 2 in the second column was crossed by two lines and hence 1 was added to the element 2 by which the new element in this column is reading 3) .All other elements which are crossed by one line remain intact (for example digit 14 in the first row remains unchanged as it is crossed by only one line). We again draw minimum number of lines crossing out all zeros in the table as shown below:
  • 25. 25 Marketing Manager Zones 1 2 3 4 5 6 A 14 3 0 5 9 10 B 1 0 13 6 8 0 C 13 19 5 4 10 0 D 0 0 7 2 10 14 E 0 2 11 1 21 17 Dummy 0 3 0 0 0 3 Note- We again see that only five lines are sufficient to cancel all zeros. We have drawn minimum number of horizontal and vertical lines with each line striking off maximum zeros. As the number of lines is less than the rows and columns, we again use step number 5 for modification. Marketing Manager Zones 1 2 3 4 5 6 A 14 3 0 4 8 10 B 1 0 13 5 7 0 C 13 19 5 3 9 0 D 0 0 7 1 9 14 E 0 2 11 0 20 17 Dummy 1 4 1 0 0 4 We again draw minimum number of lines crossing out all zeros in the table as shown below
  • 26. 26 Marketing Manager Zones 1 2 3 4 5 6 A 14 3 0 4 8 10 B 1 0 13 5 7 0 C 13 19 5 3 9 0 D 0 0 7 1 9 14 E 0 2 11 0 20 17 Dummy 1 4 1 0 0 4 As the number of lines is equal to rows or columns optimum solution has been reached and we move to step number 7 for assignment. Step 7- Proceed to job assignments as follows: Examine the rows one by one starting with the first row until a row with an exactly one zero is found. Mark the zero by enclosing it in a square indicating assignment of the task to the facility. After doing so, cross out all the zeros (if any) in that column as they cannot be used to make other assignments. Examine next the columns for any column having a single zero, starting from the first column. Mark the zero as mentioned above, crossing out the remaining zeros (if any) in that row. Repeat steps (i) and (ii) alternatively until all the zeros have been either assigned or crossed. Marketing Manager Zones 1 2 3 4 5 6 A 14 3 0 4 8 10 B 1 0 13 5 7 0 C 13 19 5 3 9 0 D 0 0 7 1 9 14 E 0 2 11 0 20 17 Dummy 1 4 1 0 0 4
  • 27. 27 Step 8-Summary of the assignment is given in the table below Marketing Manager Zone Efficiency A 3 85 B 2 83 C 6 89 D 1 91 E 4 87 : Zone 5 is left out, which can be manned by a junior manager to maximize overall efficiency of the company. 6. An airline that operates seven days in a week has time table shown below. Crews must have a minimum layover of five hours between flights. Obtain the pairing of flights that minimize layover time away from home. For any given pair the crew will be based at the city that results in smaller layover. Delhi-Jaipur Flight No Departure Arrival 101 7 A.M. 8 A.M. 102 8 A.M. 9 A.M. 103 1.30 P.M. 2.30 PM 104 6.30 P.M. 7.30 PM
  • 28. 28 Jaipur-Delhi Flight No Departure Arrival 201 8 A.M. 9.15A.M. 202 8.30 A.M. 9.45 AM 203 12 Noon 1.15PM 204 5.30 P.M. 6.45 PM For each pair also mention the place where the crew should be based. Solution: The illustration mentions about flight operating from Delhi-Jaipur and Jaipur-Delhi on a daily basis. We need to locate the crew for pair of flights where layover time (idle time) is the lowest. Since the objective is to minimize layover time between flights, we can use the Hungarian method. However before doing so we need to calculate the layover time for each pair of flights from Delhi to Jaipur and back with crew based at Delhi and similarly calculate the layover time for each pair of flights from Jaipur to Delhi and back with crew based at Jaipur. After getting the respective tables containing layover time, we select the lowest layover time out of the two tables for each pair of flights and by which get the lowest layover time table on which we need to carry out the Hungarian method.
  • 29. 29 Table consisting of Layover time in minutes for crew based in Delhi Delhi-Jaipur Return Flight Flight Number Delhi-Jaipur Flights 201 202 203 204 101 *1440 1470 1680 570 102 1380 1410 1620 510 103 1050 1080 1290 1620 104 750 780 990 1320 Note-*101 flight lands at Jaipur at 8 am. In case the crew flying on 101 wants to come back to Delhi by 201 flight which takes off from Jaipur at 8 am, the layover time between these two flights is 24 hours (8am takeoff next day from Jaipur-8am, the time it lands at Jaipur.) equals 24 hours which is 1440 minutes. Let us do one more calculation. Consider 104-202 combination. Note-104 flight lands at Jaipur at 7.30 pm and 202 flight takes off from Jaipur at 8.30 am. Therefore the layover time between this pair of flight will be 8.30 am next day and 7.30 pm earlier day, which is equal to 13 hours and in minutes 13×60= 780 minutes. Note-We have to keep in mind that the minimum layover time has to be five hours. Layover time between pair of flights is equal to the difference between takeoff of the return flight and the landing of the earlier flight. Further we avoid fractions in terms of hours we have taken minutes as the basis for the formation of the above table. Can you do the balance calculations? I am sure you will. Let us do the same exercise of calculating layover time in minutes for crew based in Jaipur for the Jaipur-Delhi sector.
  • 30. 30 Table consisting of Layover time in minutes for crew based in Jaipur Delhi-Jaipur Jaipur-Delhi Flight Number Return Flights 201 202 203 204 101 1305 **1275 1065 735 102 1365 1335 1125 795 103 1695 1665 1455 1125 104 555 525 315 1425 Note-** 202 flight lands at Delhi at 9.45 am and in case the crew on this flight wants to come back by 101 flight which departs from Delhi at 7am,the difference works out to 21 hours plus 15 minutes which is equal to 21×60+15 minutes=1275 minutes. As said before we now get the lowest layover time table from the above two tables for each pair of flights an which is as given below: Flights Flight Number 201 202 203 204 101 1305 1275 1065 570 102 1365 1335 1125 510 103 1050 1080 1290 1125 104 555 525 315 1320  means crew is based at Jaipur and if there is nomeans crew is based at Delhi. As the above table is balanced and least cost table, we can use the Hungarian method and proceed to Step number 2 for Row Minima. Step 2- Subtract the minimum element of each row of the matrix from all elements of the respective row. This step is known as Row Minima.
  • 31. 31 Flights Flight Number 201 202 203 204 101 735 705 495 0 102 855 825 615 0 103 0 30 240 75 104 240 210 0 1005 Step 3- Subtract the minimum element of each column from all the elements in that respective column. This step is known as Column Minima. Flights Flight Number 201 202 203 204 101 735 675 495 0 102 855 795 615 0 103 0 0 240 75 104 240 180 0 1005 Step4-Draw the least (minimum) number of horizontal and/or vertical lines to cover all zeros in the table. Flights Flight Number 201 202 203 204 101 735 675 495 0 102 855 795 615 0 103 0 0 240 75 104 240 180 0 1005
  • 32. 32 As the number of lines are less than the rows and columns, we move on to step number 5 for modification. Step 5- Select the smallest element not covered by the lines, subtract it from all uncovered elements including itself, add it to the elements which are crossed by two lines and reproduce other elements crossed by one line intact. Flights Flight Number 201 202 203 204 101 240 160 0 0 102 360 300 120 0 103 0 0 240 570 104 240 180 0 1500 Note- We again see that only four lines are sufficient to cancel all zeros. We have drawn minimum number of horizontal and vertical lines with each line striking off maximum zeros. As the number of lines is less than the rows and columns, we again use step number 5 for modification Flights Flight Number 201 202 203 204 101 80 0 0 0 102 200 140 120 0 103 0 0 400 730 104 240 180 0 1500 We again draw minimum number of lines crossing out all zeros in the table as shown below
  • 33. 33 Flights Flight Number 201 202 203 204 101 80 0 0 0 102 200 140 120 0 103 0 0 400 730 104 240 180 0 1500 As the number of lines is equal to rows or columns optimum solution has been reached and we move to step number 7 for assignment. Flights Flight Number 201 202 203 204 101 80 0 0 0 102 200 140 120 0 103 0 0 400 730 104 240 180 0 1500 Step 7- Proceed to job assignments as follows: Examine the rows one by one starting with the first row until a row with an exactly one zero is found. Mark the zero by enclosing it in a square indicating assignment of the task to the facility. After doing so, cross out all the zeros (if any) in that column as they cannot be used to make other assignments. Examine next the columns for any column having a single zero, starting from the first column. Mark the zero as mentioned above, crossing out the remaining zeros (if any) in that row. Repeat steps (i) and (ii) alternatively until all the zeros have been either assigned or crossed.
  • 34. 34 Flights Flight Number 201 202 203 204 101 240 160 0 0 102 360 300 120 0 103 0 0 240 570 104 240 180 0 1500 Step 8-Summary of the assignment is given in the table below Pair of Flights Crew based at Layover Time in minutes 101-202 Jaipur 1275 102-204 Delhi 510 103-201 Delhi 1050 104-203 Jaipur 315 Total 3150 Note-For pair of flight101-202 we see from the least layover time table, the crew is based at Jaipur. Remember the marking and similarly for the other pair of flight we can know where the crew will be based. 7. A travelling salesman has to visit five cities. He wishes to start from a particular city, visit each city once and then return to his starting point. The travelling cost (Rs in 000) of each city from a particular city is given below:
  • 35. 35 From City To City A B C D E A M 2 5 7 1 B 6 M 3 8 2 C 8 7 M 4 7 D 12 4 6 M 5 E 1 3 2 8 M What is the sequence of visit of the salesman to achieve the least cost route? Solution: The given assignment table is a balance assignment table and is regarding travelling salesman. We will use Hungarian method with a difference to satisfy the condition that, the salesman should visit each city before returning to the city where he started from. Note- A travelling salesman problem is typical assignment problem with two additional constraints: Salesmen should not visit the city twice until he has visited all the cities. No assignment should be made along the diagonal line. As the numbers of rows are equal to columns, we have a balanced assignment table and move on to step number 2. Step 2- Subtract the minimum element of each row of the matrix from all elements of the respective row. This step is known as Row Minima.
  • 36. 36 From City To City A B C D E A M 1 4 6 0 B 4 M 1 6 0 C 4 3 M 0 3 D 8 0 2 M 1 E 0 2 1 7 M Step 3- Subtract the minimum element of each column from all the elements in that respective column. This step is known as Column Minima. From City To City A B C D E A M 1 3 6 0 B 4 M 0 6 0 C 4 3 M 0 3 D 8 0 1 M 1 E 0 2 0 7 M Step 4- Draw the least (minimum) number of horizontal and/or vertical lines to cover all zeros in the table.
  • 37. 37 From City To City A B C D E A M 1 3 6 0 B 4 M 0 6 0 C 4 3 M 0 3 D 8 0 1 M 1 E 0 2 0 7 M As the number of lines is equal to rows or columns optimum solution has been reached and we move to step number 7 for assignment. Step 7- Proceed to job assignments as follows: Examine the rows one by one starting with the first row until a row with an exactly one zero is found. Mark the zero by enclosing it in a square indicating assignment of the task to the facility. After doing so, cross out all the zeros (if any) in that column as they cannot be used to make other assignments. Examine next the columns for any column having a single zero, starting from the first column. Mark the zero as mentioned above, crossing out the remaining zeros (if any) in that row. Repeat steps (i) and (ii) alternatively until all the zeros have been either assigned or crossed. From City To City A B C D E A M 1 3 6 0 B 4 M 0 6 0 C 4 3 M 0 3 D 8 0 1 M 1 E 0 2 0 7 M
  • 38. 38 However the solution gives the sequence A-E-A .This does not satisfy the condition that the salesman has to visit each city once before returning to the starting point. Hence we have to look at the next best solution which satisfies the above mentioned condition. This can be obtained by the next (non-zero) minimum element i.e. 1 into the solution. In the above table cost 1 occurs at three different places .Therefore we consider all these three one’s, one by one until the acceptable solution or next best solution is found to meet the above condition. Case1- We select 1 in the cell AB instead of zero assignment in the cell AE and delete row A and column B. After this step we select exclusive zero in the row or column .Except at one place i.e row number four we get exclusive zero in a row or a column. After selecting lowest element 1 in this row, in cell DE we are able to satisfy the condition. The assignment table on the above basis is as follows: From City To City A B C D E A M 1 3 6 0 B 4 M 0 6 0 C 4 3 M 0 3 D 8 0 1 M 1 E 0 2 0 7 M We have not taken other two cases as we achieved the next best solution satisfying the condition. The table below gives us the total transportation cost:
  • 39. 39 From City To City Travelling Cost INR A B 2000 B C 3000 C D 4000 D E 5000 E A 1000 Total 15000 Note-No feasible solution is obtained in case we had selected 1 in cell DC. CLOSING NOTES ON ASSIGNMENT A variety of assignment problems have been taken to give you a flavour of this topic. Students are advised to make all attempts to solve these illustrations in the CD-ROM on their own, an only when they are stuck up as a last measure refer to the solutions therein. Go step by step to achieve the optimum solution. Notes are provided in the illustrations only for guidance. You need not be a maths expert to solve the illustrations, just believe in your ability and keep a positive approach. The notes given in the illustrations are for guidance only. The notes and steps mentioned in this book as well as the CD-ROM should be used by the students to get a good insight on the diversity of the illustrations solved. While solving illustrations on their own, students are requested to mention the step in brief before writing the table below the same.